LSAT and Law School Admissions Forum

Get expert LSAT preparation and law school admissions advice from PowerScore Test Preparation.

User avatar
 Dave Killoran
PowerScore Staff
  • PowerScore Staff
  • Posts: 5925
  • Joined: Mar 25, 2011
|
#41354
Complete Question Explanation
(The complete setup for this game can be found here: lsat/viewtopic.php?t=2035)

The correct answer choice is (D)

If G does not work during the week, then from the last rule we know that H cannot work during the week. Hence, only F, I, and J can work, and the only available numerical distribution is the 2-2-1. Despite this powerful information, and the application of the fourth rule, this is still a difficult question. If you make some good inferences, but still do not see an answer choice that is immediately and obviously correct, try to use hypotheticals to solve the problem. In this instance, answer choices (A) and (C) can both be eliminated with the following hypothetical:
J94_Game_#2_#11_diagram 1.png
Note that a savvy student could use the hypothetical created in question #7 answer choice (A)—which is the same as the hypothetical above—to eliminate answer choices (A) and (C) in question #11 (remember, question #7 is a Cannot Be True question, so the four wrong answer choices are viable solutions).

Answer choices (B) and (E) can both be eliminated with the following hypothetical:
J94_Game_#2_#11_diagram 2.png

Answer choice (D) is thus proven correct
by process of elimination.

After reviewing the above information, some students still wonder why it must be that I works twice during the work week. If I only works once and F and J each work twice, then F and J are forced to “overlap” in a way that violates the fourth rule (to do otherwise causes a violation of the third rule). If you are uncertain of how this works, try to create a workable solution containing two F’s, two J’s, and one I. It cannot be done.
You do not have the required permissions to view the files attached to this post.
 satipod
  • Posts: 11
  • Joined: Apr 26, 2012
|
#22282
Hi, I had a question about problem #11. I eliminated up until I had only 2 possible answers left bu I'm wondering why the order of the firefighters couldn't be the following: IJIJI. In this case, Iman would actually work 3 times a week and answer choice D wouldn't necessarily have to be true.
User avatar
 Dave Killoran
PowerScore Staff
  • PowerScore Staff
  • Posts: 5925
  • Joined: Mar 25, 2011
|
#22283
Hey Sat,

The problem with that hypothetical is that I works three times, and that violates the second rule which states that none of the firefighters can work more than two days per week. So, we can't use that solution.

Sorry, but I hope that helps. Thanks!
 satipod
  • Posts: 11
  • Joined: Apr 26, 2012
|
#22284
Totally helpful! Don't know how I'd missed that :]
 Marie360
  • Posts: 0
  • Joined: Mar 19, 2012
|
#22285
Hi,

How do you go about solving question 11? I'm lost.

Thanks,
Marie
User avatar
 Dave Killoran
PowerScore Staff
  • PowerScore Staff
  • Posts: 5925
  • Joined: Mar 25, 2011
|
#22286
Hi Marie,

Question #11 from the June 1994 LSAT is a very tough question. Here's the explanation:

If G does not work during the week, then from the last rule we know that H cannot work during the week. Hence, only F, I, and J can work, and the only available numerical distribution is the 2-2-1. Despite this powerful information, and the application of the fourth rule, this is still a difficult question. If you make some good inferences, but still do not see an answer choice that is immediately and obviously correct, try to use hypotheticals to solve the problem. In this instance, answer choices (A) and (C) can both be eliminated with the following hypothetical:


..... ..... ..... F ..... I ..... F ..... I ..... J
..... ..... ..... M .....T .....W .....Th .....F


Note that a savvy student could use the hypothetical created in question #7 answer choice (A)—which is the same as the hypothetical above—to eliminate answer choices (A) and (C) in question #11 (remember, question #7 is a Cannot Be True question, so the four wrong answer choices are viable solutions).

Answer choices (B) and (E) can both be eliminated with the following hypothetical:


..... ..... ..... F ..... I ..... J ..... I ..... J
..... ..... ..... M .....T .....W .....Th .....F


Answer choice (D) is thus proven correct by process of elimination.

After reviewing the above information, some students still wonder why it must be that I works twice during the work week. If I only works once and F and J each work twice, then F and J are forced to “overlap” in a way that violates the fourth rule (to do otherwise causes a violation of the third rule). If you are uncertain of how this works, try to create a workable solution containing two F’s, two J’s, and one I. It cannot be done.

Please let me know if that helps. Thanks!
 AylixW
  • Posts: 14
  • Joined: Jul 11, 2012
|
#4572
Hi,

I don't understand is why I(Iman) has to work twice and why F(Fuentes) doesn't work twice. In the answer key, it says some students still wonder this and then it says the reason it doesn't work is because it violates the fourth rule. It suggests to try a workable solution containing two F's, two J's, and one I, and states that it cannot be done. I do not understand why: I F J F J doesn't work. It is not violating the fourth rule as J is working after F and it is not violating the third rule because two people aren't working consecutively. What am I missing?

Thanks for your help!!
User avatar
 Dave Killoran
PowerScore Staff
  • PowerScore Staff
  • Posts: 5925
  • Joined: Mar 25, 2011
|
#4588
Hi Aylix,

Thanks for the question. I F J F J unfortunately does conflict with the fourth rule. That rule reads: "Fuentes never works later in the week than Jackson" (italics added). That second "F" (the one on Thursday) is later in the week than the first J, and so that's what causes the issue.

Please let me know if that helps. Thanks!
 lsnewbie
  • Posts: 12
  • Joined: Aug 31, 2018
|
#57997
Hello,

I'm a little confused with this question. In looking at the answer choices, I felt like F, J and I could all work two days a week, so I didn't know which answer choice to pick.

Here is my thinking:
Option 1: FJFJI - both F and J work 2x a week, so Options A+E are out
Option 2: FIJIJ - both J and I work 2x a week, so Option C is out
Option 3: FIFIJ - both F and I work 2x a week, so B or D could both be an answer?

Also, the answers in the back list out the numerical distribution for nearly every game. Is it important we try to determine the numerical distributions for each game, especially undefined/semidefined/overloaded/underfunded games? Sorry, I haven't gotten to the part of the lessons where numerical distributions are covered at length, so maybe this question will be addressed in the videos at that time. I'm finding it hard to create inferences on top of the rules that are given in the games, and I'm not sure what the best way to go about that is. :( Does that just come with practice?

Thank you in advance for your help!
 lsnewbie
  • Posts: 12
  • Joined: Aug 31, 2018
|
#58053
Hello,
Just wondering if anyone could answer my question?
Thanks!

Get the most out of your LSAT Prep Plus subscription.

Analyze and track your performance with our Testing and Analytics Package.